La ricerca ha trovato 403 risultati

da FeddyStra
25 set 2009, 00:33
Forum: Algebra
Argomento: equazione strampalata
Risposte: 18
Visite : 6760

Pikachù ha scritto:Bevo solo acqua corrente e birra alla spina
Non ci credo... Quanto in basso si potrà ancora scendere? :cry: :cry: :cry:
da FeddyStra
24 set 2009, 16:38
Forum: Algebra
Argomento: Tra Algebra e Geometria...
Risposte: 4
Visite : 2198

Rilancio (anche se è identico al precedente). Quale punto $ P $ interno al triangolo minimizza $ \displaystyle \frac1{a_1x_1}+\frac1{a_2x_2}+\frac1{a_3x_3} $?
da FeddyStra
24 set 2009, 16:06
Forum: Algebra
Argomento: Tra Algebra e Geometria...
Risposte: 4
Visite : 2198

Siano A_1,A_2,A_3 i vertici, S=[A_1A_2A_3] l'area del triangolo e p=(a_1+a_2+a_3)/2 il semiperimetro. Si ha allora che \displaystyle a_1x_1+a_2x_2+a_3x_3=2S . Grazie alla disuguaglianza HM-AM , si ricava che \displaystyle \frac{a_1}{x_1}+\frac{a_2}{x_2}+\frac{a_3}{x_3}\ge\frac{a_1+a_2+a_3}{a_1x_1+a_...
da FeddyStra
21 set 2009, 17:47
Forum: Algebra
Argomento: somme sui binomiali
Risposte: 6
Visite : 2939

andrea91 ha scritto:Tra l'alrto, ma forse ora rompo troppo le scatole, è anche razionale
Meglio ancora: è intero. Lo puoi vedere a partire dall'identità
$ \displaystyle \sum_{k \equiv i \bmod m} \binom{n}{k} = \frac{2^n}{m} + \frac1m \sum_{j=1}^{m-1} \omega_j^{-i}(1+\omega_j)^n $.
da FeddyStra
21 set 2009, 16:01
Forum: Algebra
Argomento: somme sui binomiali
Risposte: 6
Visite : 2939

domanda bonus dato che sei stato così bravo: fissati comunque m, n come prima esiste sempre un i per cui per cui quella serie è esattamente un m-esimo del totale? Innanzi tutto, un m -esimo del totale è intero se e solo se m è una potenza di 2 minore o uguale a 2^n (che è il totale). In generale, q...
da FeddyStra
21 set 2009, 15:37
Forum: Teoria dei Numeri
Argomento: k|gcd(n,f(n))
Risposte: 9
Visite : 3547

Forse ho capito: indichiamo i numeri n costituiti da j cifre "uno" con a_j . Dunque a_j=\dfrac{10^{j+1}-1}{9} . Allora (se non sto prendendo un abbaglio, su questi argomenti non sono troppo ferrato) , se prendiamo k numeri consecutivi a_j , essi cosituiscono una classe di resto modulo k, ...
da FeddyStra
21 set 2009, 13:35
Forum: Teoria dei Numeri
Argomento: k|gcd(n,f(n))
Risposte: 9
Visite : 3547

(k,10)=1 . Esistono infiniti multipli di k formati da sole cifre 1 . Il resto della dimostrazione è ok, ma questo fatto come si dimostra? In gara non credo che si possa lasciare senza spiegazione.... Hint: pigeonhole! Il mio voleva essere solo un hint per la soluzione. Inoltre, la dimostrazione è m...
da FeddyStra
20 set 2009, 23:52
Forum: Teoria dei Numeri
Argomento: k|gcd(n,f(n))
Risposte: 9
Visite : 3547

Sia \overline{n} il numero che si ottiene rovesciando n . Distinguiamo alcuni casi: k=1 . No comment. k=2^a . \displaystyle n = \overline{k}\underbrace{00\dots00}_{\ge a}k . k=5^a . \displaystyle n = \overline{k}\underbrace{00\dots00}_{\ge a}k . 10 \mid k . Non è possibile perchè 10 \nmid \overline{...
da FeddyStra
20 set 2009, 17:37
Forum: Matematica non elementare
Argomento: sin, cos, exp
Risposte: 1
Visite : 1965

sin, cos, exp

Date le serie
  • $ \displaystyle \sum_{n=0}^{\infty} \exp(kn)x^n $
  • $ \displaystyle \sum_{n=0}^{\infty} \sin(kn)x^n $
  • $ \displaystyle \sum_{n=0}^{\infty} \cos(kn)x^n $
con $ k\in\mathbb Z $, trovare il loro raggio di convergenza e una formula chiusa che le rappresenta.
da FeddyStra
20 set 2009, 15:09
Forum: Geometria
Argomento: direi sns 1999 forse
Risposte: 12
Visite : 5489

Gauss91 ha scritto:PS: che linguaggio per le formule si usa su questo forum? Ho visto che MathML non funziona...
$ \LaTeX $
da FeddyStra
20 set 2009, 12:27
Forum: Algebra
Argomento: somme sui binomiali
Risposte: 6
Visite : 2939

Ti suggerisco un'idea abbastanza generale che si applica quando vuoi "estrarre" alcuni termini di una serie. Chiama \omega_1,\dots,\omega_m=1 le radici m -esime dell'unità. La relazione importante di cui si fa uso è \displaystyle \sum_{j=1}^{m} \omega_j^k = \begin{cases} 0 & \text{se $...
da FeddyStra
19 set 2009, 21:46
Forum: Geometria
Argomento: direi sns 1999 forse
Risposte: 12
Visite : 5489

¬[ƒ(Gabriel)³²¹º]¼+½=¾ ha scritto:temo sia sbagliato, infatti per il triangolo il punto non è il baricentro ma il punto di Fermat.
Questo non è sempre vero. Se il punto di Fermat è interno al triangolo è indubbiamente giusto, ma se esso è esterno no. Infatti è superfluo tracciare parte delle curve fuori dal triangolo.
da FeddyStra
15 set 2009, 13:16
Forum: Matematica ricreativa
Argomento: I quattro triangoli
Risposte: 42
Visite : 17314

6 a 3 contro il gatto (sì, anche un mio amico ha votato contro!).
da FeddyStra
14 set 2009, 13:12
Forum: Teoria dei Numeri
Argomento: scambiando due cifre, stessi divisori primi. O.o
Risposte: 3
Visite : 1645

Re: scambiando due cifre, stessi divisori primi. O.o

jordan ha scritto:iii) scambiando due cifre distinte...
da FeddyStra
13 set 2009, 18:49
Forum: Combinatoria
Argomento: Galileiana 2008 (8)
Risposte: 5
Visite : 2739

Ah beh io ho immaginato che fossero interi. Comunque viene un bel problema anche se sono reali. Che variabile è k? k è semplicemente una dump variable che funge da indice di sommazione. Il mio messaggio voleva semplicemente consigliare di trovare una formula chiusa per quella sommatoria e capire com...